You are on page 1of 25

Simulated Test 01

Question 1
Marks: 1
An annuitant under an Annuity Policy of Insurance company is the following person:
Choose one answer.

a. A person who receives annuity


b. A nominee.


c. An assignee.


d. None of the above.

Incorrect
Marks for this submission: 0/1.
Question 2
Marks: 1
What options does an employer have for superannuation arrangement?
Option (A) - Payment by employer
Option (B) - Funding through a trust.
Choose one answer.

a. Only (A)


b. Only (B)


c. Neither (A) nor (B)


d. Both (A) & (B)

Incorrect
Marks for this submission: 0/1.
Question 3
Marks: 1
An eligibility criterion for withdrawal from a Provident fund for purchase of a house is
___________.
Choose one answer.

a. membership of the fund for 5 years, minimum balance in members a/c Rs. 1,000,
purchase should be in favour of the member or member and spouse.

b. membership of the fund for 10 years, minimum balance in members a/c Rs. 1,000,
purchase should be in favour of the member or member and spouse.

c. membership of the fund for 5 years, minimum balance in members a/c Rs. 5,000,
purchase should be in favour of the member or member and spouse.

d. membership of the fund for 5 years, minimum balance in members a/c Rs. 1,000,
purchase should be in favour of the member or spouse.
Incorrect
Marks for this submission: 0/1.
Question 4
Marks: 1
An establishment with 100 employees, notified by Central Government and also that which
is a factory engaged in a specified industry, would be covered under Employees Provident
fund and Miscellaneous provision Act, 1952 if the number of employees, getting salary of
Rs.6500 or less, is/are _____________.
Choose one answer.

a. 20 or more


b. It will be covered even if the income of all employees is above Rs. 6500


c. 1 or more


d. More than 20

Incorrect
Marks for this submission: 0/1.
Question 5
Marks: 1
Which of the following is a defined contribution plan?
Choose one answer.

a. Leave salary


b. Gratuity


c. Unrecognized Provident Fund


d. Voluntary Retirement Scheme

Incorrect
Marks for this submission: 0/1.
Question 6
Marks: 1
As an employee Mohan has come to you with his question on superannuation plans/ Annuity
Plans to get more educated with current scenario. The term 'Employee benefits' includes
contributions in which of the following categories of schemes?
Choose one answer.

a. Non occupation disability Insurance


b. Unemployment compensation Insurance


c. Social security Scheme


d. Group Insurance Scheme

Incorrect
Marks for this submission: 0/1.
Question 7
Marks: 1
Deferred Annuity means ___________
Choose one answer.

a. purchase price could be paid in a single installment


b. annuity payments begin at the end of a selected term


c. purchase price could be paid in regular installments spread through the period of
deferment

d. All of the above.

Incorrect
Marks for this submission: 0/1.
Question 8
Marks: 1
Under EPF scheme, monthly pension after retirement is decided on _________.
Choose one answer.

a. eligible service and pensionable salary


b. pensionable service and pensionable salary


c. actual service and pensionable salary


d. None of the above.

Incorrect
Marks for this submission: 0/1.
Question 9
Marks: 1
The maximum advance a member can take from his/her Provident Fund, for major illness
involving stay in hospital for at least 1 month, is ______.
Choose one answer.

a. 12 months wage (basic + DA)


b. 24 months wage (basic + DA)


c. 6 months wage (basic + DA)


d. 36 months wage (basic + DA)

Incorrect
Marks for this submission: 0/1.
Question 10
Marks: 1
Which of the following statement/s is/are correct?

(A) Any salary surrendered by the employee to the central Government under the voluntary
Surrender of Salaries Act, 1961, will not be included while calculating his taxable income if
he is a Government employee.
(B) Any salary surrendered by the employee to the central Government under the
voluntary Surrender of Salaries Act, 1961, will not be included while calculating his taxable
income if he is a private/public sector employee.
Choose one answer.

a. only (B) is correct


b. only (A)is correct.


c. Both (A) and (B) are wrong.


d. Both (A) and (B) are correct.

Incorrect
Marks for this submission: 0/1.
Question 11
Marks: 1
U/s 10 of the Income Tax Act, Gratuity paid to a Government Employee or employees of
local authorities is exempt upto a limit of ______________.
Choose one answer.

a. Rs. 350000


b. Rs. 300000


c. Rs. 250000


d. None of the above

Incorrect
Marks for this submission: 0/1.
Question 12
Marks: 1
Deduction in respect of contributions to an annuity plan of certain pension funds under
Section 80C of IT Act is allowed to ___________
Choose one answer.

a. Any Assessee


b. Individual Assessee only


c. Individual or HUF


d. Individual, resident in India

Incorrect
Marks for this submission: 0/1.
Question 13
Marks: 1
The process of retirement planning would generally not involve ________.
Choose one answer.

a. concentrating on maximizing returns on Present / Past Investments


b. projecting individual needs and goals into the future and making sound financial plan


c. making a plan for management & disposition of assets at death


d. planning for staying physically healthy & making necessary psychological adjustment
and to plan for housing and leisure / work
Incorrect
Marks for this submission: 0/1.
Question 14
Marks: 1
The matching contribution by the Government, to the Contributory Provident Fund, for its
employees, is ____________.
Choose one answer.

a. 8.33%


b. 12%


c. 10%


d. None of the above

Incorrect
Marks for this submission: 0/1.
Question 15
Marks: 1
Which of the following statement/s is /are correct?

(i) A fixed benefit annuity is an annuity under which the insurer guaranties that at least a
defined amount of annuity benefit will be provided for each rupee applied to purchasing the
annuity.
(ii) A fixed benefit annuity is an annuity under which the insurer guaranties that a fixed
amount of annuity benefit will be provided for life.
Choose one answer.

a. Only (i) is correct.


b. Only (ii) is correct.


c. Both (i) & (ii) are correct


d. Both (i) & (i) are wrong.

Incorrect
Marks for this submission: 0/1.
Question 16
Marks: 1
Interest rate on Post Office Savings Bank is payable__________.
Choose one answer.

a. monthly


b. quarterly


c. half yearly


d. Yearly

Incorrect
Marks for this submission: 0/1.
Question 17
Marks: 1
Reciprocal of P/E ratio, E/P ratio measures___________.

Choose one answer.

a. market price of a share


b. growth ratio


c. cost of debt


d. opportunity cost of capital

Incorrect
Marks for this submission: 0/1.
Question 18
Marks: 1
Mr. Prasad is a small-scale trader. He has a welfare scheme for his few employees, for
whom he runs a small provident fund, which is yet to be recognized. He also contributes to
this fund as an employer and matches the employees contribution. He has encouraged his
employees also to open PPF accounts to save tax and set aside funds for the future. Mr.
Prasad has called you to take a look at what he has been doing and advise him on the
choice of products for providing retirement benefits to himself and his employees. Can Mr.
Prasad open joint PPF accounts with his employees?
Choose one answer.

a. A PPF account cannot be opened in joint names.


b. No, joint accounts can be held only by members of the same family or HUFs.


c. Yes, it can be opened, but the proceeds cannot be made payable to either or survivor.


d. Yes, provided the employee is the first holder.

Incorrect
Marks for this submission: 0/1.
Question 19
Marks: 1
A Provident Fund acquires its status of recognition by ___________.
Choose one answer.

a. Labour Ministry


b. SEBI


c. PF Commissioner


d. IT Authority

Incorrect
Marks for this submission: 0/1.
Question 20
Marks: 1
Which of the following statement is correct?
Choose one answer.

a. Payment under a deferred Annuity Plan starts after a specified term.


b. Payment under a deferred Annuity Plan starts on retirement.


c. Payment under a deferred Annuity Plan starts from the age of 55 years.


d. Payment under a deferred Annuity Plan starts immediately on payment of premium.

Incorrect
Marks for this submission: 0/1.
Question 21
Marks: 1
If the inflation rate is 4.9% and tax rate is 30% the required rate of return to maintain the
value of an investment will be ___________.
Choose one answer.

a. 8%


b. 9%


c. 7%


d. 10%

Incorrect
Marks for this submission: 0/1.
Question 22
Marks: 1
An auditor of an Exempted Provident Fund can be:
Choose one answer.

a. A member of FPSB India


b. A practicing Chartered Accountant


c. An employee of the Employee Provident Fund Organization


d. The auditor of the company, which is having the exempted provident fund

Incorrect
Marks for this submission: 0/1.
Question 23
Marks: 1
Business Risk is associated with
Choose one answer.

a. Economy as a whole


b. International Market


c. Nature of Enterprise


d. None of the above

Incorrect
Marks for this submission: 0/1.
Question 24
Marks: 1
Diversified Portfolio affects Systematic Risk in the following way:
Choose one answer.

a. No Impact.


b. Huge Impact.


c. Little Impact.


d. None of the above

Incorrect
Marks for this submission: 0/1.
Question 25
Marks: 1
Calculate the GDP of India with the given figures:
Consumption: $ 300 billion
Investments: $ 400 billion
Government spending: $ 300 billion
Exports: $ 150 billion
Imports: $ 200 billion
Choose one answer.

a. $1,000 billion


b. $650


c. $1,050


d. $950

Incorrect
Marks for this submission: 0/1.
Question 26
Marks: 1
Amit is eligible for gratuity as per the provisions of the Payment of Gratuity Act, 1972.
Amits employer has agreed to pay him a gratuity amount that is higher than what is
stipulated under the Act. Amit accordingly would receive a final settlement of Rs. 5,00,000/-
on retirement. What is the tax status of this amount?
Choose one answer.

a. The gratuity paid is exempt from Income Tax only to the extent of Rs.3,50,000/-.


b. The amount of gratuity payable to him cannot exceed Rs. 3,50,000/-.


c. There is no specific tax exemption for gratuity, in the Income Tax Act.


d. The Income Tax Act will only allow a maximum exemption upto 15 days wages per
completed year of service. The rest is taxable.
Incorrect
Marks for this submission: 0/1.
Question 27
Marks: 1
The investment limit in PO MIS, which can save tax under section 80C of the Income Tax
Act, 1961 to a resident individual is:

Choose one answer.

a. Min Rs. 6000/- and no upper limit


b. Min Rs. 6000/- Max 6 lakh per assessee


c. Min Rs. 6000/- Max 3 lakh per assessee


d. None of the above

Incorrect
Marks for this submission: 0/1.
Question 28
Marks: 1
For defined benefit plans, which of the following changes in actuarial assumption would
increase plan costs to an employee?

1. Early retirement without a reduction in benefits.
2. Longer life expectancy predicted
3. An increase in inflation expectations regarding labor costs.
Choose one answer.

a. 1 & 2 only


b. 1 only


c. 1, 2 & 3


d. 2 & 3 only

Incorrect
Marks for this submission: 0/1.
Question 29
Marks: 1
Sundar has been an employee of a public sector undertaking for the past 25 years and is
retiring the next year. He is eligible for gratuity as per the provisions of the Payment of
Gratuity Act, 1972. He hopes to invest the proceeds along with the PF proceeds, in order to
fund his retirement. Sundar does not receive his gratuity 45 days after his retirement. What
are his rights?
Choose one answer.

a. Sundar should receive his gratuity within 30 days after his retirement. The employer has
to pay penal interest for delay in the payment of gratuity.

b. Sundar should receive his gratuity within 60 days after his retirement. He should wait for

his settlement.

c. Sundar should receive his gratuity within 21 days after his retirement. The employer has
to pay penal interest at 15% for delay in the payment of gratuity.

d. There is no legally determined time frame for payment of gratuity. Sundar has legal
remedies if payment is not made "within reasonable time."
Incorrect
Marks for this submission: 0/1.
Question 30
Marks: 1
Investment in Senior Citizens Pension Plan qualifies for deduction u/s 80C of IT Act.
Choose one answer.

a. True


b. False


c. Only upto Rs. 70,000


d. Data insufficient

Incorrect
Marks for this submission: 0/1.
Question 31
Marks: 1
Which one of the following statements is NOT true for a defined-benefit plan.
(i) It favors older participants.
(ii) Contributions are annual & arbitrary.
(iii) It requires an actuary.
Choose one answer.

a. (i) only


b. (ii) only


c. (iii) only


d. None of the statements

Incorrect
Marks for this submission: 0/1.
Question 32
Marks: 1
No relief under section 89 of the Income Tax Act 1961 is admissible if taxable Gratuity is in
respect of services rendered for less than ________ years.
Choose one answer.

a. 15


b. 5


c. 9


d. None of the above.

Incorrect
Marks for this submission: 0/1.
Question 33
Marks: 1
Which amongst the following is true about the interest received from Recognized Provident
Fund (RPF)?
Choose one answer.

a. Not treated as income in the year of credit.


b. Not treated as income if the rate is equal to/less than 9.5%. Rate over and above this is
taxable

c. Fully exempt from tax.


d. None of the above

Incorrect
Marks for this submission: 0/1.
Question 34
Marks: 1
A person wants to earn Rs. 5000 a month when the interest rate is 6%. What is the capital
required?
Choose one answer.

a. Rs. 1000000


b. Rs. 10000


c. Rs. 500000


d. Rs. 50000

Incorrect
Marks for this submission: 0/1.
Question 35
Marks: 1
A person invests Rs. 70000 on the 1st of April for 5 year. The amount accumulated at 8% is
_______
Choose one answer.

a. Rs. 443515


b. Rs. 434515


c. Rs. 443155


d. Rs. 434155

Incorrect
Marks for this submission: 0/1.
Question 36
Marks: 1
Changing the weights of assets in the portfolio at regular intervals is called _____.
Choose one answer.

a. Diversifying


b. Rebalancing


c. Concentrating


d. None of the above

Incorrect
Marks for this submission: 0/1.
Question 37
Marks: 1
One of the biggest reasons to undertake portfolio evaluation is _________.
Choose one answer.

a. poor planning


b. no analysis


c. changing needs of the individual


d. young age of the person for whom the plan ismade

Incorrect
Marks for this submission: 0/1.
Question 38
Marks: 1
If there is a requirement of capital protection then _________ instrument will not be used.
Choose one answer.

a. Cash equivalents


b. Government securities


c. Mid cap shares


d. Bonds

Incorrect
Marks for this submission: 0/1.
Question 39
Marks: 1
The number of members covered in a group _______.
Choose one answer.

a. Are fixed


b. Remain the same


c. Fluctuate


d. None of the above

Incorrect
Marks for this submission: 0/1.
Question 40
Marks: 1
If you want to tweak the cover on a policy and customize then this will not be possible in
________
Choose one answer.

a. Group Insurance


b. Individual insurance


c. Term Insurance


d. Unit linked policy

Incorrect
Marks for this submission: 0/1.
Question 41
Marks: 2
per month and Dearness Allowance is 40% of Basic Salary for retirement benefits. He
retired from his job on January 1, 2009 and shifted to his village. He is entitled to the
following benefits at the time of retirement.
Gratuity = Rs.98000.
Pension from January 1, 2009 = Rs. 2000 per month.
Payment from recognized PF = Rs. 300000.
Encashment of earned leave for 150 days = Rs. 36000.
He was entitled to 40 days leave for every completed year of service. He got 50% of his
pension commuted in lump sum w.e.f. March 1, 2009 and received Rs. 120000 as
commuted pension.
Gaurav contributes Rs. 900 per month to RPF to which his employer contributes an equal
amount. With these details you are required to help him with the following questions related
to previous year 2008-09. What amount will he get as the tax exemption under the
Payment of Gratuity Act, 1972?
Choose one answer.

a. Rs. 63,000/-


b. Rs. 67,846/-


c. Rs. 35,000/-


d. Rs. 3,50,000/-

Incorrect
Marks for this submission: 0/2.
Question 42
Marks: 2
Vishal is working with Amex Ltd since October 1, 1995. He is entitled to a basic salary of Rs.
6,000 per month. Dearness Allowance is 40% of Basic Salary for retirement benefits. He
retired from his job on January 1, 2009. He is entitled to the following benefits at the time
of retirement.
Gratuity = Rs. 98,000.
Pension from January 1, 2009 = Rs. 2,000 per month.
Payment from recognized PF = Rs. 3,00,000.
Encashment of earned leave for 150 days = Rs. 36,000.
He was entitled to 40 days leave for every completed year of service. He got 50% of his
pension commuted in lump sum w.e.f April 1, 2009 and received Rs. 1,20,000 as commuted
pension. Vishal contributes Rs. 900 per month to RPF to which his employer contributes an
equal amount. What will be the amount of un-commuted pension for Vishal that will form
part of his total income for the A.Y. 2009-10?
Choose one answer.

a. NIL


b. Rs. 4,000


c. Rs. 6,000


d. Rs. 1,000

Incorrect
Marks for this submission: 0/2.
Question 43
Marks: 2
Ashok receives leave encashment of Rs. 2,00,000 on account of accumulated leave of 240
days. He has completed 24 years of service and his last 10 months average monthly salary
is Rs. 10,000. What is the taxable portion of leave encashment for Ashok if he was entitled
to 40 days leave for every year of service?
Choose one answer.

a. Rs. 1,00,000


b. Rs. 2,00,000


c. Nil


d. Rs. 2,50,000

Incorrect
Marks for this submission: 0/2.
Question 44
Marks: 2
Consider a portfolio of two investments viz. A & B. The sum total of volatility of A and B
respectively, represented by standard deviation of the two investments, will be equal to the
volatility of the portfolio as a whole if _________________.
Choose one answer.

a. A and B have a correlation of Zero


b. A and B have a correlation of 1


c. The portfolio is equally divided between A and B


d. The return on the portfolio is equal to the sum of returns of A and B.

Incorrect
Marks for this submission: 0/2.
Question 45
Marks: 2
Amit works as a seasonal employee in an establishment covered under Gratuity Act. His
date of joining is 01 Apr 1978, and works till 31 Mar 2008. His wages (Basic + DA) at the
time of retirement is Rs. 10000 per month. Work out his gratuity entitlement.
Choose one answer.

a. Rs. 115385


b. Rs. 80769


c. Rs. 173077


d. Rs. 350000

Incorrect
Marks for this submission: 0/2.
Question 46
Marks: 2
An insurance firm has to create and sell a policy to cover the risk of partial disability to
workmen in a chemical factory. Which of the following is not a relevant factor in pricing the
policy?
Choose one answer.

a. The ability of the insurance firm to deploy the premium and earn positive investment
returns on the funds.

b. The ability of the workmen to bear the cost of the policy.


c. The extent of probable disability created due to the work environment and its impact on
workmen.s earning capability.

d. The willingness of the management of the firm to bear part of the costs of the premium.


Incorrect
Marks for this submission: 0/2.
Question 47
Marks: 2
Raj joins ABC Ltd. on 25th April 1968. ABC has 30 employees. If he is expected to retire on
15th November 2008, and his last drawn salary for gratuity purposes is Rs. 26,000 per month,
what amount of gratuity would be exempted from tax on account of Rajs gratuity entitlement if
ABC Ltd is covered under the payment of Gratuity Act, 1972?
Choose one answer.

a. Rs. 6,00,000.


b. Rs. 5,20,000.


c. Rs. 3,50,000.


d. Rs. 3,12,000

Incorrect
Marks for this submission: 0/2.
Question 48
Marks: 2
Kulkarni is running an unrecognised Provident Fund, because he has a smaller number of
employees in his firm. Which of the following is true about the tax status of such a fund?
Choose one answer.

a. Only the employees. contribution is eligible for a rebate under Section 80C.


b. Kulkarni.s contribution as the employer is exempt from tax in his books.


c. The interest received on such unrecognised PFs is taxable every year.


d. Accumulated employees contribution is taxable at the time of retirement

Incorrect
Marks for this submission: 0/2.
Question 49
Marks: 2
The savings of a person is Rs. 8000 per month. What should the figure turn out to be each
year person has to accumulate Rs 25 lakh in 15 years at an earnings rate of 7%?
Choose one answer.

a. Rs. 98486


b. Rs. 99486


c. Rs. 97486


d. Rs. 96486

Incorrect
Marks for this submission: 0/2.
Question 50
Marks: 2
Mr. Roy has been an employee of a public sector undertaking for the past 25 years and is
retiring the next year. He is eligible for gratuity as per the provisions of the Payment of
Gratuity Act, 1972. He hopes to invest the proceeds along with the PF proceeds, in order to
fund his retirement. Prior to his wedding,Mr. Roy had nominated his mother to receive his
gratuity. Is his wife eligible to receive the gratuity proceeds in the event of his death?
Choose one answer.

a. All nominations made prior to the employee acquiring a family are invalid once an
employee has acquired a family.

b. As long as a valid nomination is in place, the payment of gratuity will be made only to
such nominees.

c. The payment of gratuity will always be according to the registered will, irrespective of
the nominations.

d. The information is not sufficient for giving an answer.

Incorrect
Marks for this submission: 0/2.
Question 51
Marks: 2
Mr. Rajans investment portfolio comprises Rs.2 lakh in equity, Rs.5 lakh in debt and Rs. 1
lakh in his bank current account. Over one year the returns on equity and debt are 5% and
12%. At the end of the year to maintain his current asset allocation, he needs to
_____________
Choose one answer.

a. Do nothing


b. He needs to move Rs, 10000/- from equity and Rs. 60000/- from debt to cash.


c. He needs move Rs.7500/- to equity from debt and Rs. 8750/-to cash from debt


d. He needs to invest Rs. 70000/- in debt and equity.

Incorrect
Marks for this submission: 0/2.
Question 52
Marks: 2
Which of the following financial transactions / events would affect net worth of your client?

(A) Repayment of a loan using fund from a savings account.
(B) Purchase of car which is 75% financed with 25% down payment.
(C) The Nifty is appreciating, and the client is holding Nifty Indexed Mutual Fund.
(D) Interest rate increases and the client holds substantial bond portfolio.
Choose one answer.

a. (C) & (D)


b. (B) & (C)


c. (A), (C) & (D)


d. All of the above

Incorrect
Marks for this submission: 0/2.
Question 53
Marks: 2
A 10 year 8.0% bond (Face Value- Rs.1000, interest payable semi-annually) maturing 6
years from today is available at a yield to maturity of 6.0%. It is likely to be priced at
_______________.
Choose one answer.

a. Rs. 1100


b. Rs. 1149


c. Rs. 1168


d. Rs. 1498

Incorrect
Marks for this submission: 0/2.
Question 54
Marks: 2
Raykar is an accomplished Financial Planner and is also an expert on derivatives and high
yielding bonds. He understands client requirements well and is able to come up with
appropriate portfolio restructuring ideas for clients. He believes in quickly moving clients
from one investment to another through a dynamic process of research and
recommendations. What according to the Rules relating to the Code of Ethics is the most
applicable in this case?
Choose one answer.

a. He does not violate the Rules if he explains to the client the reasons and is able to show
that the moves are appropriate to the client.

b. He does not violate the Rules since he conducts and has access to research and advises
on products relevant to clients based on an understanding of their requirements.

c. He does not violate the Rules since he is an acknowledged expert and knows what is best
for his clients.

d. He violates the Rules as it amounts to active churning of client portfolios.

Incorrect
Marks for this submission: 0/2.
Question 55
Marks: 2
Mrs. & Mr. Arora are aged 55 and 58 years respectively. Both expect to work till they turn
65. Their only goal is to fund their retirement. Which of the following is likely to be an
appropriate asset allocation strategy for them?
Choose one answer.

a. 10% sectoral equity, 20% diversified equity, 30% long-term debt, and 40% medium term
debt

b. 20% Sectoral equity, 60% diversified equity, 20% long-term debt


c. 30% Sectoral equity, 30% diversified equity, 40% cash/ liquid investments.


d. 80% long-term debt, 20% medium term debt

Incorrect
Marks for this submission: 0/2.
Question 56
Marks: 2
If the post tax rate of return on an investment is 8% and the inflation rate is 5% the real
rate of return is_______________.
Choose one answer.

a. 3.5%


b. 3.0%


c. 2.86%


d. -3.0%

Incorrect
Marks for this submission: 0/2.
Question 57
Marks: 2
Sahil, age 43, can refinance Rs. 114042 at a 20-year rate for 7% and will incur closing cost
of 3% of the mortgage amount to be financed in the new mortgage balance. What will be
his new EMI on the mortgage under the circumstances to achieve his objective of no debt at
retirement (age 60)?
Choose one answer.

a. Rs. 781.49


b. Rs. 957.56


c. Rs. 980.57


d. Rs. 986.29

Incorrect
Marks for this submission: 0/2.
Question 58
Marks: 2
Any possible occurrence which may have a negative financial implication can be plotted on a
graph with X axis measuring the frequency (low-high) and Y axis measuring the financial
impact (low-high). You can view the classification in four quadrants. Quadrant I - Low
frequency, Low Impact Quadrant II - Low frequency, High Impact Quadrant III High
frequency, High Impact Quadrant IV - High frequency, Low Impact It would not be practical
to purchase insurance for events falling in _________________.
Choose one answer.

a. Quadrant I & IV


b. Quadrant I, II & IV


c. Quadrant I, III & IV


d. Quadrant III

Incorrect
Marks for this submission: 0/2.
Question 59
Marks: 2
Which of the following is a correct interpretation of the Rules of Conduct pertaining to the
Ethic of Confidentiality?
Choose one answer.

a. A Member must when requested by the client, provide to a person authorized by the
client, all original documents prepared or received by the Member in undertaking the
advisory task.


b. A Member owes to the Member.s partners or co-owners a responsibility to act in good
faith (expectations of confidentiality) only while in business together, not thereafter.

c. The Member shall maintain the same standards of confidentiality to employers as to
clients.

d. Under no circumstance, will any Member divulge any information or knowledge

regarding the FPSB India or its members that they may know or be exposed to.
Incorrect
Marks for this submission: 0/2.
Question 60
Marks: 2
Data available for Arvind and Neha is:

Assets: Cash = Rs. 50,000, Savings A/C = Rs. 50,000, MMMF (Money market mutual funds)
= Rs. 2,00,000,
Bonds = Rs. 2,50,000, Fixed Deposit = Rs. 1,50,000, Marketable Stocks = Rs. 5,00,000,
Equity based Mutual Funds = Rs. 30,000, Real Estate Vacant Plot = Rs. 5,00,000
Yearly Expenses = Rs. 3,60,000, Yearly Income Net Of Taxes = Rs. 6,00,000.
Liabilities: Car loan = Rs. 2,50,000, House Loan = Rs. 8,00,000.
Yearly Expenses include EMI for car and House Loan.
Find Basic Liquidity Ratio of the Couple
Choose one answer.

a. 3.33


b. 10


c. 26.67


d. 15.5

Incorrect
Marks for this submission: 0/2.
Question 61
Marks: 4
Nitish is a 30 year-old self employed youth and has been using the PPF account to
accumulate Rs. 30,000 per year, for his future needs. The PPF account provides a
compounded return of 8% p.a. He does not have a clear view yet on financial goals and
needs but has been saving as a habit, for the last 5 years. Nitish is willing to look at a
lifestyle after retirement that fits into a fixed Rs. 3,00,000 per annum spend, for an
estimated 15 years. What is the spending opportunity for Nitish, at the time of his
retirement at 60 years, given his saving and assuming a rate of 6% on his funds after
retirement? (Assumption: All computations for interest spend and savings compound
annually, assuming beginning of the period investment. Answers to be rounded off to the
nearest 5 rupees).
Choose one answer.

a. Rs. 20,81,345


b. Rs. 18,54,425


c. Rs. 14,26,890


d. Rs. 24,94,570

Incorrect
Marks for this submission: 0/4.
Question 62
Marks: 4
Ajay has worked in two companies viz. A Co. and B Co. He retires from A Co. on Nov 30,
1990 (salary at the time of retirement Rs.2,600) and receives Rs. 22,000 as gratuity out of
which Rs. 20,000 were exempted. He also retires from B Co. on Dec 10, 2008 after 28 years
and 8 months of service and receives Rs. 2.90 lakh as death-cum-retirement gratuity. His
average basic salary drawn from B Co. for the preceding 10 months ending on Nov 30, 2008
is Rs. 18,200 per month. Further he receives Rs. 1,000 per month as DA, 80% of which
forms a part of his retirement benefits and 6% commission on turnover achieved by him.
Total turnover achieved by him during 10 months ending Nov 30, 2008 is Rs. 2 lakh. What
is the gratuity exempted from tax for Ajay for AY 2009-10?
Choose one answer.

a. Rs. 290000


b. Rs. 2,80,000


c. Rs. 3,30,000


d. Rs. 2,62,800

Incorrect
Marks for this submission: 0/4.
Question 63
Marks: 4
Vinita was recently divorced and has two children. The divorce decree requires that she pay
1/3 of the college tuition cost for her children. The tuition cost is currently Rs. 15,000 per
year and has been increasing at 7% per year. Her son and daughter are 12 and 16
respectively and will attend college for four years beginning at age 18. How much should
she save each month, beginning today for the next five years to finance education for both
the children (in nearest rupee)? Assume that her after-tax rate of return will be 9% and that
general inflation has been 4% p.a.
Choose one answer.

a. Rs. 750


b. Rs. 745


c. Rs. 2,235


d. Rs. 2,500

Incorrect
Marks for this submission: 0/4.
Question 64
Marks: 4
Nirav, aged 30, wants to retire at 45. He wants to maintain his present living standard. He
spends
Rs. 3,25,000 a year. He is expected to live upto 85. Inflation is to be assumed at 4% p.a
and expected returns are 7% p.a. What is the nest egg required by Nirav at his age 45 and
what amount should he save every year end to meet his plan? His present investments are
Rs 10,00,000.
Choose one answer.

a. Nest egg and savings required will be Rs. 1,41,82,828 and Rs. 4,54,606 respectively.


b. Nest egg and savings required will be Rs. 1,47,73,065 and Rs. 7,40,530 respectively.


c. Nest egg and savings required will be Rs. 1,27,73,065 and Rs. 4,38,300 respectively.


d. Nest egg and savings required will be Rs. 2,51,00,065 and Rs. 5, 41,093 respectively.

Incorrect
Marks for this submission: 0/4.
Question 65
Marks: 4
Laxman is an NRI who has been working in the US for the past 5 years. He is aged 40. He
has been saving Rs. 8 lakh per annum for the past 5 years and hopes to save the same
amount for the next 10 years that he plans to live in the US. He would like to return to India
10 years from now. The inflation-adjusted monthly income requirement for Laxman, as
estimated by the planner, is Rs. 80,000 in the year in which he returns to India. It is
estimated that inflation would remain at an average of 3% p.a for the next 30 years. His life
expectancy is placed at 70 years. However if the estimated spend per month, for his family
is Rs. 90,000 p.m., and the rate of inflation is 4% p.a, how long will his savings last?
Assumption: His investments will earn a rate of interest of 6% pa. Compounding to be done
on annuity certain basis throughout the problem.
Choose one answer.

a. The funds will last until Laxman is 81 years old.


b. The funds will not be adequate to fund his estimated life span of 70 years.


c. The funds will last until Laxman is 72 years old.


d. The funds will exhaust before Laxman is 71 years old.

Incorrect
Marks for this submission: 0/4.
Question 66
Marks: 4
Ahmeds current annual expenditure is Rs.100000/-. He is 30 years old and expects to retire
at age 55. His annual expenses are estimated to rise by 6% p.a. and his life expectancy is
75 years. His post retirement annual expenses are estimated to be 80% of his pre-
retirement expenses. What will be his expenses onthe first year of his retirement?
Choose one answer.

a. Rs. 3,47,425/-


b. Rs. 3,43,350/-


c. Rs. 4,29,187/-


d. Rs. 4,02,350

Incorrect
Marks for this submission: 0/4.
Question 67
Marks: 4
Mr. Nirav an affluent broker wants to retire at 45. He wants to maintain his present living
standard. He spends Rs.3,25,000/- a year. He is expected to live upto 85. Inflation is to be
assumed at 4% and expected returns are 7% p.a. How can he achieve this? He is at present
30 years old. What is the nest egg required at age 45 and what amount shall he save every
year to meet his plan? His present investments are Rs.10,00,000/-.(Assumption: All
computations for interest, spend and savings compound annually, assuming beginning of
the period investment).
Choose one answer.

a. Nest egg and savings required will be Rs.1,47,73,065/- and Rs.7,40,530/- respectively.



b. Nest egg and savings required will be Rs.1,41,82,819/- and Rs.4,24,865/- respectively.



c. Nest egg and savings required will be Rs.1, 27,73,065/- and Rs.4,38,300/- respectively.



d. Nest egg and savings required will be Rs.2, 51,00,065/- and Rs.5, 41,093/- respectively


Incorrect
Marks for this submission: 0/4.
Question 68
Marks: 4
Ms.Rekha is 45 years old and plans to retire at 50. Her life expectancy is 70 years.
Ms.Sushma, her Financial Planner, estimates that her client will require Rs.45,000/- in the
first month after retirement. Inflation rate is 4% p.a. and the rate of return is 6% p.a. What
will be the savings per year required in order to meet this?
Choose one answer.

a. Rs. 15,90,000/- (Approx)


b. Rs. 14,80,000/- (Approx)


c. Rs. 16,90,000/- (Approx)


d. Rs. 12,40,000/- (Approx)

Incorrect
Marks for this submission: 0/4.
Question 69
Marks: 4
Aditi is 30 years old. She deposits 25000 at the beginning of each year in deferred annuity
scheme as a part of her retirement planning. How much will be in her account after 25
years if her savings earn 9.5% compound annual interest?
Choose one answer.

a. Rs. 2474985


b. Rs. 2487216


c. Rs. 2414854


d. Rs. 2497857

Incorrect
Marks for this submission: 0/4.
Question 70
Marks: 4
Seema has been an employee of a public sector undertaking (covered under the payment of
Gratuity Act, 1972) for the past 20 (completed) years and is retiring on the 31st of
December this year. Her monthly salary at retirement is expected to be Rs. 20000. The
amount of gratuity that she will receive is ________.
Choose one answer.

a. Rs. 241245


b. Rs. 242308


c. Rs. 243579


d. Rs. 241940

Incorrect
Marks for this submission: 0/4.
Question 71
Marks: 4
Mrs. Shah retired from Ace Manufacturing Co. Ltd. Mumbai on 31/12/2008. Ace is covered
under the Payment of Gratuity Act, 1972. She served for 30 years and 9 months. Ace paid
her a Gratuity of Rs.400000. Her monthly basic salary at the time of retirement was Rs.
9000 p.m. and Dearness Allowance was Rs. 4000 p.m. House Rent Allowance was Rs. 1500
p.m. Mrs. Shah lives in an ownership flat. Compute: Taxable amount of Gratuity & Taxable
amount of HRA of Mrs. Shah.
Choose one answer.

a. Gratuity: Rs. 160000; HRA: Rs.20000


b. Gratuity: Rs. 157500; HRA: Rs.16000


c. Gratuity: Rs. 170000; HRA: Rs.18000


d. Gratuity: Rs. 167500; HRA: Rs.18000

Incorrect
Marks for this submission: 0/4.
Question 72
Marks: 4
Sumeet, aged 25, plans to retire at age 55. His life expectancy is 75. His current annual
expenditure is Rs. 250000. He estimates no reduction in his expenses post-retirement. If
interest rate is expected to be 8.5% and inflation is 5% p.a. estimate how much will he
have to save per annum in order to achieve his target, provided he does not wish to leave
an estate?
Choose one answer.

a. Rs. 129696


b. Rs. 125054


c. Rs. 117154


d. Rs. 120963

Incorrect
Marks for this submission: 0/4.
Question 73
Marks: 4
Mr. Jay Mehta wants to start saving for his son Aryans (age 5 years) higher education in
the USA. He will spend six years at the college in the USA beginning at age 18. The present
cost structure of higher education in the USA is:
Year 1 Rs. 15 lakh
Year 2 Rs. 17 lakh
Year 3 Rs. 19 lakh
Year 4 Rs. 21 lakh
Year 5 Rs. 23 lakh
Year 6 Rs. 25 lakh
The cost of education is increasing by 6% every year. If Jay starts investing in an equity
Mutual Fund (15% pa) how much he need to deposit at the end of each year to pay for
Aryans higher educational requirements? Assume that educational expenses are withdrawn
at the beginning of each year and that the last deposit will be made at the beginning of the
last year of Aryans college education.
Choose one answer.

a. Rs. 4,37,553/-


b. Rs. 4,74,700/-


c. Rs. 5,03,186/-


d. Rs. 5,45,906/-

Incorrect
Marks for this submission: 0/4.
Question 74
Marks: 4
Which of the following is a tort of negligence?
Choose one answer.

a. Mr. Joy was playing golf. He swings a new golf club on the fairway and the head of the
club flies off, and hit another golfer who was standing 20 feet away.

b. Mr. Vishal takes medication that he knows makes him drowsy and then proceeds to
drive. He gets into an accident injuring the passengers in another car.

c. Mrs. Jaya locks Ms. Rani in a room to prevent her from leaving the building.


d. Mrs. Priti experienced a sudden surge of chest pain while driving, which causes her to
lose control of her car and hit another car.
Incorrect
Marks for this submission: 0/4.
Question 75
Marks: 4
Sachin is aged 25 years and has annual expenditure of 2 lakhs. If the inflation is 6 % for 1st
10 years , 7 % for next 15 years and 5 % for next 5 years, calculate his expenditure at
retirement ( 55 yrs ) at 80 % replacement ?
Choose one answer.

a. 457126


b. 475126


c. 451726


d. 1261222

You might also like